negation of answer
When negating A, what words do you change to negate? Would you: (1) change the word can to cannot...
BritneyS on August 24 at 12:12AM
  • October 1996 LSAT
  • SEC4
  • Q8
1
Reply
How is D not the answer ?
Medication is NOT always more effective in treating patients with mental disorders than is psycho...
Nativeguy on April 18, 2023
  • October 1996 LSAT
  • SEC4
  • Q8
1
Reply
I need more help rationalizing the correct answ...
Would a correct answer also be "A large proportion of mail is damaged in transit"? For me I wo...
AndrewArabie on February 15, 2023
  • October 1996 LSAT
  • SEC4
  • Q19
6
Replies
Why is A correct
Can someone explain why A is correct?
Amandaccs on January 5, 2023
  • October 1996 LSAT
  • SEC4
  • Q18
1
Reply
Why B? Why not D?
I don't understand how B doesn't resolve the paradox. I thought D wouldn't resolve the paradox...
AndrewArabie on December 3, 2022
  • October 1996 LSAT
  • SEC4
  • Q26
2
Replies
Was stuck between B and D
Why is B wrong? That would throw the argument apart if some of the government workers earn more t...
Tyler808 on July 13, 2022
  • October 1996 LSAT
  • SEC4
  • Q16
2
Replies
Please help
Could you please explain why the answer is C?
yckim2180 on January 21, 2022
  • October 1996 LSAT
  • SEC4
  • Q14
1
Reply
Correct Answer
Please explain why A, is the correct answer? I'm not sure if I made the question harder than it r...
Anthony-Resendes on October 1, 2020
  • October 1996 LSAT
  • SEC4
  • Q13
1
Reply
Could someone please explain the correct answer?
Could someone please explain the correct answer? Thanks
jingjingxiao11111@gmail.com on May 27, 2020
  • October 1996 LSAT
  • SEC4
  • Q26
1
Reply
can you diagram the question
can someone diagram the actual question's passage?
hatemz on February 23, 2020
  • October 1996 LSAT
  • SEC4
  • Q15
1
Reply
answer?
A?
Lucas on January 21, 2020
  • October 1996 LSAT
  • SEC4
  • Q13
1
Reply
Hello
Why is A not the correct answer?
baahmed7860 on January 15, 2020
  • October 1996 LSAT
  • SEC4
  • Q1
1
Reply
Answer D versus Answer C
Can you please explain why answer choice D is better than answer choice C? I ended up choosing an...
Samantha-Alexis on January 11, 2020
  • October 1996 LSAT
  • SEC4
  • Q21
1
Reply
Why would A not work here?
Aren't A and D saying very similar things? They both seem to refer to the fact that people can be...
MACZ on December 28, 2019
  • October 1996 LSAT
  • SEC4
  • Q21
1
Reply
Why D?
Hi Could you please explain why it's D Thank you
farnoushsalimian on December 2, 2019
  • October 1996 LSAT
  • SEC4
  • Q11
1
Reply
I also have the same question as the person abo...
Please answer.
Patricia-Adekunle on August 13, 2019
  • October 1996 LSAT
  • SEC4
  • Q21
1
Reply
Diagramming
I got the right answer but Can you diagram answer choice B so I can check whether I diagrammed it...
Virginia_61092 on June 19, 2019
  • October 1996 LSAT
  • SEC4
  • Q15
2
Replies
Quantified
In the first sentence, should "nearly all" be considered to mean "most"? Or should it be diagramm...
stephd on January 28, 2016
  • October 1996 LSAT
  • SEC4
  • Q19
1
Reply
Help
Is this "must be true" question? Then, please explain why choice (e) is not the answer. Thanks,
Batman on July 2, 2014
  • October 1996 LSAT
  • SEC4
  • Q10
2
Replies